Economics Final

Lakukan tugas rumah & ujian kamu dengan baik sekarang menggunakan Quizwiz!

A recent report by the U.S. Centers for Disease Control and Prevention (CDC), published in the CDC's Morbidity and Mortality Weekly Report, studied the effect of an increase in the price of beer on the incidence of new cases of sexually transmitted disease in young adults. In particular, the researchers analyzed the responsiveness of gonorrhea cases to a tax-induced increase in the price of beer. The report concluded that "the ... analysis suggested that a beer tax increase of $0.20 per six-pack could reduce overall gonorrhea rates by 8.9%." Assume that a six-pack costs $5.90 before the price increase. Use the midpoint method to determine the percent increase in the price of a six-pack, and then calculate the cross-price elasticity of demand between beer and incidence of gonorrhea. According to your estimate of this cross-price elasticity of demand, are beer and gonorrhea complements or substitutes?

-2.7

Fishing for sablefish has been so intensive that sablefish were threatened with extinction. After several years of banning such fishing, the government is now proposing to introduce tradable vouchers, each of which entitles its holder to a catch of a certain size. Explain how uncontrolled fishing generates a negative externality and how the voucher scheme may overcome the inefficiency created by this externality.

An individual fisherman makes decisions about how much fish to catch based on his or her marginal benefit and marginal cost. However, the marginal social cost of fishing is greater than the fishing industry's marginal cost, since catching fish reduces the number of fish that can reproduce and so imposes an external cost on other fishermen. Since an individual fisherman does not take this external cost into account in deciding how much to fish, there will be too much fishing compared to what would be socially optimal. Assuming that the number of vouchers allocated to all fishermen corresponds to the socially optimal quantity of fish caught, the voucher scheme could achieve efficiency: it will limit the size of the total catch to the socially optimal quantity. And since the vouchers are tradable, fishermen who are more efficient (can operate at a lower cost) will buy vouchers from less efficient ones, so only the most efficient fishermen will operate.

Don owns a small concrete-mixing company. His fixed cost is the cost of the concrete-batching machinery and his mixer trucks. His variable cost is the cost of the sand, gravel, and other inputs for producing concrete; the gas and maintenance for the machinery and trucks; and his workers. He is trying to decide how many mixer trucks to purchase. He has estimated the costs shown in the accompanying table based on estimates of the number of orders his company will receive per week. For each level of fixed cost, calculate Don's total cost for producing 20, 40, and 60 orders per week. If Don is producing 20 orders per week, how many trucks should he purchase and what will his average total cost be? Answer the same questions for 40 and 60 orders per week.

Quantity 20 40 60 of trucks orders orders orders 2 $8,000 8,800 9,200 3 $11,000 10,800 11,600 4 $18,000 17,800 16,400 b. Don should choose the number of trucks that minimizes average total cost for each level of output. Given this, Don should buy two trucks if he is producing 20 orders per week. His average total cost per order will be $400. He should buy three trucks if he is producing 40 orders per week. His average total cost per order will then be $270. He should buy four trucks if he is producing 60 orders per week. His average total cost per order will then be $273.

Scott finds that the higher the price of orange juice, the more money he spends on orange juice. Does that mean that Scott has discovered a Giffen good?

Scott has not necessarily discovered a Giffen good. For a good to be a Giffen good, the quantity demanded of the good has to increase as its price rises. However, Scott has only found that the amount of money he spends on purchases of orange juice has increased as its price rises. For instance, suppose the price of orange juice were to rise from $5 per half-gallon to $10 per half-gallon, and as a result Scott reduces the quantity of orange juice demanded from 3 half-gallons to 2 half-gallons. This means that orange juice is not a Giffen good, since the quantity demanded decreases as the price rises. However, Scott's spending on orange juice would have increased from $5 × 3 = $15 to $10 × 2 = $20. So an increase in spending on a good as its price rises need not necessarily imply that the good is a Giffen good.

You have the information shown in the accompanying table about a firm's costs. Complete the missing data. (in downloads)

The accompanying table contains the complete cost data. The total cost of producing one unit of output is the total cost of producing zero units of output plus the mar- ginal cost of increasing output from zero to one, and so forth. The average total cost is just the total cost divided by output. Since the total cost of producing zero output is $20, the variable cost is TC − $20. The average variable cost is then just the vari- able cost divided by output. - - $40.00 $20.00 25.00 15.00 22.00 15.33 21.50 16.50 22.00 18.00

The U.S. government would like to help the American auto industry compete against foreign automakers that sell trucks in the United States. It can do this by imposing an excise tax on each foreign truck sold in the United States. The hypothetical pre-tax demand and supply schedules for imported trucks are given in the accompanying table. In the absence of government interference, what is the equilibrium price of an imported truck? The equilibrium quantity? Illustrate with a diagram. Assume that the government imposes an excise tax of $3,000 per imported truck. Illustrate the effect of this excise tax in your diagram from part a. How many imported trucks are now purchased and at what price? How much does the foreign automaker receive per truck? Calculate the government revenue raised by the excise tax in part b. Illustrate it on your diagram. How does the excise tax on imported trucks benefit American automakers? Whom does it hurt? How does inefficiency arise from this government policy?

The equilibrium price without government interference is $30,000 and the equilibrium quantity is 300,000, as shown by point E in the accompanying diagram. b. The effect of the excise tax is illustrated in the diagram: a tax of $3,000 per truck puts a wedge between the price paid by consumers, or the demand price ($31,000), and the price received by producers, or the supply price ($28,000). The quantity bought and sold is 200,000 trucks. The foreign automaker receives $28,000 per truck (after tax). c. Since 200,000 trucks are sold, and the government earns a tax of $3,000 on each truck, the total tax revenue is 200,000 × $3,000 = $600 million. This is the shad- ed area in the diagram. d. The excise tax leads to a rise in the price of imported trucks. Since American trucks are substitutes for imported trucks, the effect of the tax is to increase the demand for American-made trucks, which leads to a higher price for them. As a result, buyers of both domestic and foreign trucks pay higher prices because of the tax on foreign trucks. Inefficiency arises because some mutually beneficial trans- actions no longer occur due to the higher prices for trucks caused by the tax.

A perfectly competitive firm has the following short-run total cost: Market demand for the firm's product is given by the following market demand schedule: Calculate this firm's marginal cost and, for all output levels except zero, the firm's average variable cost and average total cost. There are 100 firms in this industry that all have costs identical to those of this firm. Draw the short-run industry supply curve. In the same diagram, draw the market demand curve. What is the market price, and how much profit will each firm make?

Screenshot 2 #9

In the United States, each state government can impose its own excise tax on the sale of cigarettes. Suppose that in the state of North Texarkana, the state government imposes a tax of $2.00 per pack sold within the state. In contrast, the neighboring state of South Texarkana imposes no excise tax on cigarettes. Assume that in both states the pre-tax price of a pack of cigarettes is $1.00. Assume that the total cost to a resident of North Texarkana to smuggle a pack of cigarettes from South Texarkana is $1.85 per pack. (This includes the cost of time, gasoline, and so on.) Assume that the supply curve for cigarettes is neither perfectly elastic nor perfectly inelastic. Draw a diagram of the supply and demand curves for cigarettes in North Texarkana showing a situation in which it makes economic sense for a North Texarkanan to smuggle a pack of cigarettes from South Texarkana to North Texarkana. Explain your diagram. Draw a corresponding diagram showing a situation in which it does not make economic sense for a North Texarkanan to smuggle a pack of cigarettes from South Texarkana to North Texarkana. Explain your diagram. Suppose the demand for cigarettes in North Texarkana is perfectly inelastic. How high could the cost of smuggling a pack of cigarettes go until a North Texarkanan no longer found it profitable to smuggle? Still assume that demand for cigarettes in North Texarkana is perfectly inelastic and that all smokers in North Texarkana are smuggling their cigarettes at a cost of $1.85 per pack, so no tax is paid. Is there any inefficiency in this situation? If so, how much per pack? Suppose chip-embedded cigarette packaging makes it impossible to smuggle cigarettes across the state border. Is there any inefficiency in this situation? If so, how much per pack?

a. In the accompanying figure, the demand for cigarettes in North Texarkana is relatively inelastic. So most of the $2.00 tax is borne by consumers, who pay an after-tax price of $2.95. Since it would cost $2.85 to purchase and smuggle a pack from South Texarkana ($1.00 price per pack + $1.85 smuggling cost per pack), this diagram illustrates a situation in which a North Texarkanan would be better off smuggling rather than purchasing cigarettes in North Texarkana. b. In the accompanying diagram, the demand in North Texarkana is less inelastic. As a result, consumers pay an after-tax price of $2.50. In this case, it does not make economic sense to smuggle. c. As shown in the accompanying diagram, if the demand for cigarettes in North Texarkana is perfectly inelastic, the demand curve is a vertical line and all of the tax is borne by consumers. In that case, the after-tax price paid by North Texarkanans is $3.00. So the cost of smuggling could go as high as $1.99, and North Texarkanans would still be better off smuggling; at a cost of $2.00 to smug- gle, they would be indifferent between smuggling and purchasing their cigarettes in their home state. d. Since demand is perfectly inelastic, the same number of cigarettes are transacted after the tax is imposed compared to before the tax is imposed. But there is still an inefficiency incurred in this situation despite the fact that no tax is paid and no transactions are discouraged: it is the $1.85 that is spent to smuggle a pack of cigarettes. This is the value of resources spent to evade the tax that consumers would have preferred to spend on other goods and activities. If technology elimi- nates smuggling altogether, there is no inefficiency. Because demand is perfectly inelastic, no transactions are discouraged by the tax, and all of the surplus lost by consumers is captured by the government as tax revenue.

The U.S. government is considering reducing the amount of carbon dioxide that firms are allowed to produce by issuing a limited number of tradable allowances for carbon dioxide (CO2) emissions. In an April 25, 2007, report, the U.S. Congressional Budget Office (CBO) argues that "most of the cost of meeting a cap on CO2 emissions would be borne by consumers, who would face persistently higher prices for products such as electricity and gasoline ... poorer households would bear a larger burden relative to their income than wealthier households would." What assumption about one of the elasticities you learned about in this chapter has to be true for poorer households to be disproportionately affected?

lFour ptooireor hnouseholds to be disproportionately affected by an increase in energy prices, it is necessary that those households spend a larger share of their income on energy products than wealthier households. In other words, as income rises, the quantity of energy products demanded has to increase less than proportionately. So the CBO must think that the income elasticity of demand for energy products, although positive, is less than 1: energy products are income-inelastic. In fact, this is just what the CBO report says: "lower-income households tend to spend a larger fraction of their income than wealthier households do and . . . energy products account for a bigger share of their spending."

You own and operate a bike store. Each year, you receive revenue of $200,000 from your bike sales, and it costs you $100,000 to obtain the bikes. In addition, you pay $20,000 for electricity, taxes, and other expenses per year. Instead of running the bike store, you could become an accountant and receive a yearly salary of $40,000. A large clothing retail chain wants to expand and offers to rent the store from you for $50,000 per year. How do you explain to your friends that despite making a profit, it is too costly for you to continue running your store?

You yearly accounting profit is: $200,000 (total revenue) − $100,000 (cost of bikes) − $20,000 (electricity, taxes, and other expenses) $80,000 (accounting profit) But not renting the store to the retail chain is an opportunity cost, and not being able to make $40,000 as an accountant is also an opportunity cost, so your yearly economic profit is: $80,000 (accounting profit) − $40,000 (opportunity cost of your time) − $50,000 (opportunity cost of not renting the store) − $10,000 (economic profit) So although you make an accounting profit each year, you would be better off rent- ing the store to the large chain and becoming an accountant yourself, since your opportunity cost of continuing to run your own store is too high.

Walmart is the world's largest retailer. As a consequence, it has sufficient bargaining power to push its suppliers to lower their prices so it can honor its slogan of "Always Low Prices" for its customers. Is Walmart acting like a monopolist or monopsonist when purchasing goods from suppliers? Explain. How does Walmart affect the consumer surplus of its customers? The producer surplus of its suppliers? Over time, what is likely to happen to the quality of products produced by Walmart suppliers?

a monopsonist

This diagram illustrates your local electricity company's natural monopoly. It shows the demand curve for kilowatt-hours (kWh) of electricity, the company's marginal revenue (MR) curve, its marginal cost (MC) curve, and its average total cost (ATC) curve. The government wants to regulate the monopolist by imposing a price ceiling. If the government does not regulate this monopolist, which price will it charge? Illustrate the inefficiency this creates by shading the deadweight loss from monopoly. If the government imposes a price ceiling equal to the marginal cost, $0.30, will the monopolist make profits or lose money? Shade the area of profit (or loss) for the monopolist. If the government does impose this price ceiling, do you think the firm will continue to produce in the long run? If the government imposes a price ceiling of $0.50, will the monopolist make a profit, lose money, or break even?

a. $0.80 b. If the government imposes a price ceiling of $0.30, the quantity demanded is 10,000. The monopolist will incur a loss equal to the shaded rectangle in the accompanying figure. Since the firm is incurring a loss, in the long run it will exit the market. c. If the government imposes a price ceiling of $0.50, the quantity demanded is 8,000. The price equals the monopolist's average total cost, and so the firm will make zero profit.

A recent study determined the following elasticities for Volkswagen Beetles: The supply of Beetles is elastic. Based on this information, are the following statements true or false? Explain your reasoning. A 10% increase in the price of a Beetle will reduce the quantity demanded by 20%. An increase in consumer income will increase the price and quantity of Beetles sold.

a. A 10% increase in the price of a Beetle will reduce the quantity demanded by 20%. This statement is true because the price elasticity of demand for the Beetles is 2. The price change is 10% and the demand change is 20%. So, it is elastic since the demand change is greater than the price change of 10%. b. An increase in consumer income will increase the price and quantity of Beetles sold. Since price elasticity of demand is greater than 1, total revenue will go down. This statement is false because an increase in customer or consumer income will increase the quantity of Beetles sold but because both the price and quantity will increase despite the price elasticity of the demand the total revenue will increase not decrease

The accompanying table lists the cross-price elasticities of demand for several goods, where the percent quantity change is measured for the first good of the pair, and the percent price change is measured for the second good. Explain the sign of each of the cross-price elasticities. What does it imply about the relationship between the two goods in question? Compare the absolute values of the cross-price elasticities and explain their magnitudes. For example, why is the cross-price elasticity of McDonald's burgers and Burger King burgers less than the cross-price elasticity of butter and margarine? Use the information in the table to calculate how a 5% increase in the price of Pepsi affects the quantity of Coke demanded. Use the information in the table to calculate how a 10% decrease in the price of gasoline affects the quantity of SUVs demanded.

a. A negative cross-price elasticity of demand implies that the two goods are gross complements. So air-conditioning units and kilowatts of electricity are gross complements, as are sport-utility vehicles and gasoline. A positive cross-price elasticity of demand implies that the two goods are gross substitutes. So Coke and Pepsi are gross substitutes, as are McDonald's and Burger King burgers as well as butter and margarine. b. The larger (and positive) the cross-price elasticity of demand is, the more closely the two goods are gross substitutes. Since the cross-price elasticity of butter and margarine is larger than the cross-price elasticity of McDonald's burgers and Burger King burgers, butter and margarine are closer gross substitutes than are McDonald's and Burger King burgers. Similarly, the greater (and negative) the cross-price elasticity of demand is, the more strongly the two goods are gross complements. c. A cross-price elasticity of 0.63 implies that a 1% increase in the price of Pepsi would increase the quantity of Coke demanded by 0.63%. Therefore, a 5% increase in the price of Pepsi would increase the quantity of Coke demanded by five times as much, that is, by 5 × 0.63% = 3.15%. d. A cross-price elasticity of −0.28 implies that a 1% fall in the price of gasoline would increase the quantity of SUVs demanded by 0.28%. Therefore, a 10% fall in the price of gasoline would increase the quantity of SUVs demanded by 10 times as much, that is, by 10 × 0.28% = 2.8%.

For each of the following situations, decide whether the bundle Lakshani is considering is optimal or not. If it is not optimal, how could Lakshani improve her overall level of utility? That is, determine which good she should spend more on and which good should she spend less on. Lakshani has $200 to spend on sneakers and sweaters. Sneakers cost $50 per pair, and sweaters cost $20 each. She is thinking about buying 2 pairs of sneakers and 5 sweaters. She tells her friend that the additional utility she would get from the second pair of sneakers is the same as the additional utility she would get from the fifth sweater. Lakshani has $5 to spend on pens and pencils. Each pen costs $0.50 and each pencil costs $0.10. She is thinking about buying 6 pens and 20 pencils. The last pen would add five times as much to her total utility as the last pencil. Lakshani has $50 per season to spend on tickets to football games and tickets to soccer games. Each football ticket costs $10 and each soccer ticket costs $5. She is thinking about buying 3 football tickets and 2 soccer tickets. Her marginal utility from the third football ticket is twice as much as her marginal utility from the second soccer ticket.

a. Al has increasing marginal utility of economics classes. Each additional class adds more to his total utility than the previous class. b. Al has constant marginal utility of volume of music. His total utility increases by 5 utils for each additional notch of volume, so his marginal utility is constant at 5 utils. c. Al has diminishing marginal utility of Friends episodes. Although additional episodes increase his total utility, they do so less and less. That is, his marginal utility declines. d. Al has diminishing marginal utility of marshmallows. For a certain range, addi- tional marshmallows add to his total utility, so total utility increases. But total utility increases by less and less. In fact, total utility eventually begins to decline. In other words, his marginal utility becomes smaller and smaller and eventually becomes negative.

In each of the following situations, describe the substitution effect and, if it is significant, the income effect. In which direction does each of these effects move? Why? Ed spends a large portion of his income on his children's education. Because tuition fees rise, one of his children has to withdraw from college. Homer spends much of his monthly income on home mortgage payments. The interest on his adjustable-rate mortgage falls, lowering his mortgage payments, and Homer decides to move to a larger house. Pam thinks that Spam is an inferior good. Yet as the price of Spam rises, she decides to buy less of it.

a. As tuition fees rise, college education becomes relatively more expensive compared to other goods. So Ed decides to substitute away from college education and toward other goods. This is the substitution effect. Since tuition takes up a large portion of his income, the income effect will also be significant. As tuition rises, Ed, in a real sense, becomes poorer: the purchasing power of his income falls. As a result, he will buy less of all normal goods. College education is a normal good, so the income effect also moves in the direction of less college education. The effects reinforce each other. b. As mortgage payments decrease, large homes become cheaper compared to other goods. So Homer will substitute toward buying a larger home. This is the substitu- tion effect. Since he spends much of his income on mortgage payments, the fall in mortgage rates also increases his income in a real sense: the purchasing power of his income is now higher. This implies that Homer will now buy more of all nor- mal goods. Housing is a normal good, so the income effect will also move in the direction of more housing. The effects reinforce each other. c. As its price rises, Spam becomes relatively more expensive compared to other goods. So Pam will substitute away from Spam and toward other goods. This is the substitution effect. Spam probably does not account for a large portion of Pam's income, so the income effect is likely to be negligible. However, we do know that since Spam is an inferior good, the income effect would make Pam want to con- sume more of it. As the price of Spam rises, Pam is now, in a real sense, poorer: her income buys fewer goods. Since she is now poorer, she will buy more inferior goods—that is, the income effect will lead her to buy more Spam. However, we know that overall she buys less Spam as its price rises, so the substitution effect outweighs the income effect.

Changes in the price of key commodities have a significant impact on a company's bottom line. For virtually all companies, the price of energy is a substantial portion of their costs. In addition, many industries—such as those that produce beef, chicken, high-fructose corn syrup and ethanol—are highly dependent on the price of corn. In particular, corn has seen a significant increase in price. Explain how the cost of energy can be both a fixed cost and a variable cost for a company. Suppose energy is a fixed cost and energy prices rise. What happens to the company's average total cost curve? What happens to its marginal cost curve? Illustrate your answer with a diagram. Explain why the cost of corn is a variable cost but not a fixed cost for an ethanol producer. When the cost of corn goes up, what happens to the average total cost curve of an ethanol producer? What happens to its marginal cost curve? Illustrate your answer with a diagram.

a. Energy required to keep a company operating regardless of how much output is produced represents a fixed cost, such as the energy costs of operating office buildings, factories, and stores that must be maintained independent of the amount of output produced. In addition, energy is a variable cost because produc- ing more output almost always requires using more energy. b. When fixed costs increase, so will average total costs. The average total cost curve will shift upward. In panel (a) of the accompanying diagram, this is illustrated by the movement of the average total cost curve from its initial position, ATC1, to its new position, ATC2. The marginal cost curve is not affected if the variable costs do not change. So the marginal cost curve remains at its initial position, MC. c. Since corn is an input into the production of ethanol, producing a larger quantity of ethanol requires a larger quantity of corn, making corn a variable cost. d. When variable costs increase, so do average total costs and marginal costs. Both curves will shift upward. In panel (b) of the accompanying diagram, the move- ment of the average total cost curve is illustrated by the shift from its initial posi- tion, ATC1 to its new position, ATC2. The movement of the marginal cost curve is illustrated by the shift from its initial position, MC1, to its new position, MC2.

The accompanying table shows Tanisha's and Ari's individual marginal benefit of different amounts of street cleanings per month. Suppose that the marginal cost of street cleanings is constant at $9 each. If Tanisha had to pay for street cleaning on her own, how many street cleanings would there be? Calculate the marginal social benefit of street cleaning. What is the optimal number of street cleanings? Consider the optimal number of street cleanings. The last street cleaning of that number costs $9. Is Tanisha willing to pay for that last cleaning on her own? Is Ari willing to pay for that last cleaning on his own?

a. If Tanisha had to pay for the street cleaning on her own,she would pay for the street to be cleaned once: her individual marginal benefit of the first cleaning, $10, exceeds the marginal cost of $9. However, she would not pay for more than one: her mar- ginal benefit of the second cleaning is $6, less than the marginal cost of $9. b. The accompanying table shows the marginal social benefit of street cleaning. The optimal number of street cleanings is 2: the marginal social benefit of the second cleaning is $10, which exceeds the marginal cost of $9. A third cleaning would be inefficient because its marginal social benefit is $3, less than the marginal cost of $9. Marginal social benefit $18 $10 $3 c. Tanisha on her own would be willing to pay only $6 (her individual marginal ben- efit) for the second cleaning. Ari on his own would be willing to pay only $4 (his individual marginal benefit) for the second cleaning. So neither would be indi- vidually willing to pay for the second cleaning.

The Collegetown movie theater serves 900 students and 100 professors in town. Each student's willingness to pay for a movie ticket is $5. Each professor's willingness to pay is $10. Each will buy only one ticket. The movie theater's marginal cost per ticket is constant at $3, and there is no fixed cost. Suppose the movie theater cannot price-discriminate and charges both students and professors the same price per ticket. If the movie theater charges $5, who will buy tickets and what will the movie theater's profit be? How large is consumer surplus? If the movie theater charges $10, who will buy movie tickets and what will the movie theater's profit be? How large is consumer surplus? Assume the movie theater can price-discriminate between students and professors by requiring students to show their student ID, charging students $5 and professors $10, how much profit will the movie theater make? How large is consumer surplus?

a. If the movie theatre charges $5 per ticket ,both students and professors will buy tickets. The movie theater will sell to 1,000 customers (students and professors), at a price of $5 each. Since the movie theater's cost per ticket is $3, its profit is $2 per ticket for a total profit of 1,000 × $2 = $2,000. Students will experience no consumer surplus, but each of the 100 professors will experience consumer sur- plus of $10 − $5 = $5 for a total consumer surplus of 100 × $5 = $500. b. If the movie theater charges $10 per ticket, only professors will buy tickets. The movie theater will sell to 100 customers (professors) at a price of $10 each. Since the movie theater's cost per ticket is $3, its profit is $7 per ticket for a total profit of 100 × $7 = $700. Students experience no consumer surplus since they do not buy any tickets. Each of the 100 professors experiences no consumer surplus since the price is equal to their willingness to pay. So consumer surplus is $0. c. If the movie theater charges students a price of $5, it sells 900 tickets at a profit of $5 − $3 = $2 each for a profit from selling to students of 900 × $2 = $1,800. Charging professors $10, it sells 100 tickets at a profit of $10 − $3 = $7 each for a profit from selling to professors of 100 × $7 = $700. So the theater's total profit is $1,800 + $700 = $2,500. Since each customer is charged exactly his or her willing- ness to pay, there is no consumer surplus.

Assess the following four tax policies in terms of the benefits principle versus the ability-to-pay principle. A tax on gasoline that finances maintenance of state roads An 8% tax on imported goods valued in excess of $800 per household brought in on passenger flights Airline-flight landing fees that pay for air traffic control A reduction in the amount of income tax paid based on the number of dependent children in the household.

a. This tax is based on the benefits principle, since the people who use the state's roads will be the ones paying the gasoline tax. b. This tax is based on the ability-to-pay principle, since the people paying the tax will presumably be individuals who buy expensive items abroad and then import them on passenger flights. c. This tax is based on the benefits principle, since the airlines pay the landing fee and are also the beneficiary of air traffic control services. d. This deduction is based on the ability-to-pay principle. People who have more dependent children in their household will have higher expenses and so are less able to pay a given amount of income taxes, other things equal.

Software has historically been an artificially scarce good—it is nonrival because the cost of replication is negligible once the investment to write the code is made, but software companies make it excludable by charging for user licenses. But then open-source software emerged, most of which is free to download and can be modified and maintained by anyone. Discuss the free-rider problem that might exist in the development of open-source software. What effect might this have on quality? Why does this problem not exist for proprietary software, such as the products of a company like Microsoft or Adobe? Some argue that open-source software serves an unsatisfied market demand that proprietary software ignores. Draw a typical diagram that illustrates how proprietary software may be underproduced. Put the price and marginal cost of software on the vertical axis and the quantity of software on the horizontal axis. Draw a typical demand curve and a marginal cost curve (MC) that is always equal to zero. Assume that the software company charges a positive price, P, for the software. Label the equilibrium point and the efficient point.

a. In principle, the developers of open-source software are not strictly monitored. Some developers may shirk and write poor code in the hope that others in the development community will correct their mistakes. A free-rider problem is cre- ated because individual developers are not held responsible for their code, poten- tially resulting in poor quality. Microsoft and Adobe, however, are responsible for the quality of their software; they risk losing business and profits if their product is substandard. So company management enforces quality-control measures that mitigate the free-rider problem. b. The accompanying diagram shows a demand curve, D, and a marginal cost curve that is constant and always equal to zero, MC. The equilibrium is at point EMKT , with a quantity, QMKT , that is lower than the efficient quantity, QOPT . (decreasing)

A profit-maximizing business incurs an economic loss of $10,000 per year. Its fixed cost is $15,000 per year. Should it produce or shut down in the short run? Should it stay in the industry or exit in the long run? Suppose instead that this business has a fixed cost of $6,000 per year. Should it produce or shut down in the short run? Should it stay in the industry or exit in the long run?

a. In the short run, the business should produce. If it shuts down, the short-run annual loss will be $15,000, its fixed cost; but if it produces, the loss will be only $10,000. In the long run, the business should exit the industry because it is incur- ring a loss. b. In the short run, the business should shut down. If it shuts down, the short-run loss will be $6,000, its fixed cost; if it continues to produce, the loss will be $10,000. In the long run, the firm should exit the industry because it is incurring a loss.

In each of the following examples, explain whether the decision is rational or irrational. Describe the type of behavior exhibited. Kookie's best friend likes to give her gift cards that Kookie can use at her favorite stores. Kookie, however, often forgets to use the cards before their expiration date or loses them. Kookie, though, is careful with her own cash. In 2010, the Panera Bread company opened a store in Clayton, Missouri, that allowed customers to pay any amount they like for their orders; instead of prices, the store listed suggested donations based on the cost of the goods. All profits went to a charitable foundation set up by Panera. In 2011, the store was pleased with the success of the program. Rick has just gotten his teaching degree and has two job offers. One job, replacing a teacher who has gone on leave, will last only two years. It is at a prestigious high school, and he will be paid $35,000 per year. He thinks he will probably be able to find another good job in the area after the two years are up but isn't sure. The other job, also at a high school, pays $25,000 per year and is virtually guaranteed for five years; after those five years, he will be evaluated for a permanent teaching position at the school. About 75% of the teachers who start at the school are hired for permanent positions. Rick takes the five-year position at $25,000 per year.

a. Kookie is behaving irrationally, engaging in mental accounting. By losing the dol- lars incorporated in the gift cards but not losing her cash, she is valuing a dollar in a gift card less than a dollar in her wallet. b. Customers of Panera Bread are behaving rationally, exhibiting concerns about fair- ness. They are willing to reduce their own economic payoffs in order to be fair to Panera Bread and the beneficiaries of the Panera charitable foundation. c. Rick is behaving rationally, exhibiting risk aversion. Although he could potentially make more money by teaching at the prestigious high school for two years, this is a riskier proposition than taking the job that is guaranteed for five years.

Magnificent Blooms is a florist specializing in floral arrangements for weddings, graduations, and other events. Magnificent Blooms has a fixed cost associated with space and equipment of $100 per day. Each worker is paid $50 per day. The daily production function for Magnificent Blooms is shown in the accompanying table. Calculate the marginal product of each worker. What principle explains why the marginal product per worker declines as the number of workers employed increases? Calculate the marginal cost of each level of output. What principle explains why the marginal cost per floral arrangement increases as the number of arrangements increases?

a. MPL, shown in the accompanying table for the five workers, is the change in out- put resulting from the employment of one additional worker per day. MPL falls as the quantity of labor increases due to the principle of diminishing returns. b. The marginal cost, MC, of floral arrangements is the change in total cost divided by the change in output. So, to compute MC, we first need to compute total cost, TC = FC + VC, as shown in the table. MC per floral arrangement is also shown in the table. MC increases as output increases due again to the principle of diminish- ing returns.

Bob produces Blu-ray movies for sale, which requires a building and a machine that copies the original movie onto a Blu-ray. Bob rents a building for $30,000 per month and rents a machine for $20,000 a month. Those are his fixed costs. His variable cost per month is given in the accompanying table. Calculate Bob's average variable cost, average total cost, and marginal cost for each quantity of output. There is free entry into the industry, and anyone who enters will face the same costs as Bob. Suppose that currently the price of a Blu-ray is $25. What will Bob's profit be? Is this a long-run equilibrium? If not, what will the price of Blu-ray movies be in the long run?

a. Screenshot b. When the price is $25, Bob will sell 8,000 DVDs per month and make a profit of $78,000. If there is free entry into the industry, this profit will attract new firms. As firms enter, the price of DVDs will eventually fall until it is equal to the mini- mum average total cost. Here, the average total cost reaches its minimum of $13.83 at 6,000 DVDs per month. So the long-run price of DVDs will be $13.83.

Suppose that De Beers is a single-price monopolist in the diamond market. De Beers has five potential customers: Raquel, Jackie, Joan, Mia, and Sophia. Each of these customers will buy at most one diamond—and only if the price is just equal to, or lower than, her willingness to pay. Raquel's willingness to pay is $400; Jackie's, $300; Joan's, $200; Mia's, $100; and Sophia's, $0. De Beers's marginal cost per diamond is $100. The result is a demand schedule for diamonds as follows: Calculate De Beers's total revenue and its marginal revenue. From your calculation, draw the demand curve and the marginal revenue curve. Explain why De Beers faces a downward-sloping demand curve and why the marginal revenue from an additional diamond sale is less than the price of the diamond. Suppose De Beers currently charges $200 for its diamonds. If it lowers the price to $100, how large is the price effect? How large is the quantity effect? Add the marginal cost curve to your diagram from part a and determine which quantity maximizes De Beers's profit and which price De Beers will charge.

a. Screenshot 3 #6 b. De Beers is the only producer of diamonds, so its demand curve is the market demand curve. And the market demand curve slopes downward: the lower the price, the more customers will buy diamonds. c. If De Beers lowers the price sufficiently to sell one more diamond, it earns extra revenue equal to the price of that one extra diamond. This is the quantity effect of lowering the price. But there is also a price effect: lowering the price means that De Beers also has to lower the price on all other diamonds, and that lowers its revenue. So the marginal revenue of selling an additional diamond is less than the price at which the additional diamond can be sold. d. If the price is $200, then De Beers sells to Raquel, Jackie, and Joan. If it lowers the price to $100, it will also sell a diamond to Mia. The price effect is that De Beers loses $100 (the amount by which it lowered the price) each from selling to Raquel, Jackie, and Joan. So the price effect lowers De Beers's revenue by 3 × $100 = $300. The quantity effect is that De Beers sells one more diamond (to Mia), at $100. So the quantity effect is to raise De Beers's revenue by $100. e. The marginal cost (MC) curve is constant at $100, as shown in the diagram. Marginal revenue equals marginal cost at a quantity of 2 diamonds. So De Beers will sell 2 diamonds at a price of $300 each.

Download Records decides to release an album by the group Mary and the Little Lamb. It produces the album with no fixed cost, but the total cost of creating a digital album and paying Mary her royalty is $6 per album. Download Records can act as a single-price monopolist. Its marketing division finds that the demand schedule for the album is as shown in the accompanying table. Calculate the total revenue and the marginal revenue per album. The marginal cost of producing each album is constant at $6. To maximize profit, what level of output should Download Records choose, and which price should it charge for each album? Mary renegotiates her contract and will be paid a higher royalty per album. So the marginal cost rises to be constant at $14. To maximize profit, what level of output should Download Records now choose, and which price should it charge for each album?

a. Screenshot 4 #9 b. If the marginal cost of each album is $6, DR will maximize profit by producing 4,000 albums, since for each album up to 4,000, marginal revenue is greater than marginal cost. For any further albums, marginal cost would exceed marginal revenue. Producing 4,000 albums, Download Records will charge $14 for each album. c. If the marginal cost of each album is $14, Download Records will maximize profit by producing 2,000 albums, and it will charge $18 per album.

The accompanying table shows six consumers' willingness to pay (his or her individual marginal benefit) for one MP3 file copy of a Jay-Z album. The marginal cost of making the file accessible to one additional consumer is constant, at zero. Consumer Individual marginal benefit Adriana $2 Bhagesh 15 Chizuko 1 Denzel 10 Emma 5 Frank 4 What would be the efficient price to charge for a download of the file? All six consumers are able to download the file for free from a file-sharing service, Pantster. Which consumers will download the file? What will be the total consumer surplus to those consumers? Pantster is shut down for copyright law infringement. In order to download the file, consumers now have to pay $4.99 at a commercial music site. Which consumers will download the file? What will be the total consumer surplus to those consumers? How much producer surplus accrues to the commercial music site? What is the total surplus? What is the deadweight loss from the new pricing policy?

a. Since the marginal cost of delivering the good to one additional consumer is zero, the efficient price would be zero. b. Since each of the six consumers has a marginal benefit greater than zero, all six will download the file. Adriana's individual consumer surplus will be $2, Bhagesh's $15, Chizuko's $1, Denzel's $10, Emma's $5, and Frank's $4. The total consumer surplus is therefore $2 + $15 + $1 + $10 + $5 + $4 = $37 c. At a price of $4.99, Bhagesh, Denzel, and Emma will download the file. Bhagesh's individual consumer surplus will be $10.01, Denzel's $5.01, and Emma's $0.01. So total consumer surplus is $10.01 + $5.01 + $0.01 = $15.03. Producer surplus is 3 × $4.99 = $14.97. So total surplus is $15.03 + $14.97 = $30. This is $7 less than in part b. So the deadweight loss from making the good artificially scarce is $7.

The production of agricultural products like wheat is one of the few examples of a perfectly competitive industry. In this question, we analyze results from a study released by the U.S. Department of Agriculture about wheat production in the United States back in 2013. The average variable cost per acre planted with wheat was $127 per acre. Assuming a yield of 44 bushels per acre, calculate the average variable cost per bushel of wheat. The average price of wheat received by a farmer in 2013 was $7.58 per bushel. Do you think the average farm would have exited the industry in the short run? Explain. With a yield of 44 bushels of wheat per acre, the average total cost per farm was $4.80 per bushel. The harvested acreage for rye (a type of wheat) in the United States increased from 242,000 in 2010 to 306,000 in 2013. Using the information on prices and costs here and in parts a and b, explain why this might have happened.

a. Since the yield is 50 bushels per acre, we know that producing 50 bushels of wheat is associated with an average variable cost of $107. So the production of 1 bushel of wheat is associated with an average variable cost of $107/50 bushels = $2.14 per bushel. b. We would not expect the average farm to have exited the industry in the short run because the price it received for wheat, $2.65 per bushel, was greater than the average variable cost of production, $2.14 per bushel. c. The average farm would have exited the industry in the long run because the price it received per bushel was less than the average total cost of production. The farm was incurring an economic loss by operating. So a decline in the harvested acreage of wheat should have been expected after 1998.

The government is involved in providing many goods and services. For each of the goods or services listed, determine whether it is rival or nonrival in consumption and whether it is excludable or nonexcludable. What type of good is it? Without government involvement, would the quantity provided be efficient, inefficiently low, or inefficiently high? Street signs Amtrak rail service Regulations limiting pollution A congested interstate highway without tolls A lighthouse on the coast

a. Street signs are nonrival in consumption (if I make use of a street sign, that does not reduce your opportunity to use it) and nonexcludable (no one can prevent another person from making use of a street sign). So street signs are a public good. Because of the free-rider problem, the quantity provided privately would be inefficiently low. b. Amtrak rail service is rival in consumption (if I consume a seat, you cannot) and excludable (you cannot consume the service if you do not have a ticket). Although Amtrak rail service is a private good, it creates a positive externality in the form of reduced road and air traffic congestion. The market would provide an inefficiently low level of passenger rail service, so there is a justification for government inter- vention to support Amtrak. c. Regulations limiting pollution are nonrival in consumption (my benefit from these regulations is not diminished by your benefit) and nonexcludable (people cannot be selectively excluded from benefiting from these regulations—that is, excluded from breathing clean air or drinking clean water). So these regulations are a public good. Because of the free-rider problem, the privately provided quan- tity of these regulations would be inefficiently low. d. A congested interstate highway without tolls is rival in consumption (if I use the highway, I create a negative externality for you—more congestion; that is, I reduce your benefit from the highway) but nonexcludable (drivers can use the highway without paying for access). So the highway is a common resource. Because of nonexcludability, a free-rider problem exists, and the privately provided quantity of highways would be inefficiently low. e. A lighthouse is nonrival in consumption (if I use the lighthouse to steer my boat away from rocks, you can still use the same lighthouse) and nonexcludable (boats cannot selectively be made to pay for the services provided by the lighthouse). So the lighthouse is a public good. Because of the free-rider problem, the privately provided quantity would be inefficiently low.

Voluntary environmental programs were extremely popular in the United States, Europe, and Japan in the 1990s. Part of their popularity stems from the fact that these programs do not require legislative authority, which is often hard to obtain. The 33/50 program started by the Environmental Protection Agency (EPA) is an example of such a program. With this program, the EPA attempted to reduce industrial emissions of 17 toxic chemicals by providing information on relatively inexpensive methods of pollution control. Companies were asked to voluntarily commit to reducing emissions from their 1988 levels by 33% by 1992 and by 50% by 1995. The program actually met its second target by 1994. As in Figure 16-3, draw marginal benefit curves for pollution generated by two plants, A and B, in 1988. Assume that without government intervention, each plant emits the same amount of pollution, but that at all levels of pollution less than this amount, plant A's marginal benefit of polluting is less than that of plant B. Label the vertical axis "Marginal benefit to individual polluter" and the horizontal axis "Quantity of pollution emissions." Mark the quantity of pollution each plant produces without government action. Do you expect the total quantity of pollution before the program was put in place to have been less than or more than the optimal quantity of pollution? Why? Suppose the plants whose marginal benefit curves you depicted in part a were participants in the 33/50 program. In a replica of your graph from part a, mark targeted levels of pollution in 1995 for the two plants. Which plant was required to reduce emissions more? Was this solution necessarily efficient? What kind of environmental policy does the 33/50 program most closely resemble? What is the main shortcoming of such a policy? Compare it to two other types of environmental policies discussed in this chapter.

a. Two lines decreasing MBb on top meet at x-axis b. We should expect that the total quantity of pollution before the plan was adopted was above the optimal quantity because pollution generates a negative externality. When the negative externality is not internalized or regulated, it results in higher market activity than is optimal. c. The accompanying diagram shows the targeted level of emissions in 1995, Q1995. Both firms had to reduce their emissions by the same amount. This was not nec- essarily efficient: since at the quantity Q1995, plant B had a higher marginal benefit of pollution, the situation could have been more efficient by allowing plant B to pollute a little more and asking plant A to reduce its emissions more. d. The 33/50 program set an environmental standard. The main shortcoming of this type of policy is that its inflexibility often prevents pollution reductions from being achieved at the lowest cost. Tradable permits and emissions taxes are more flexible policies than an environmental standard. These policies help to achieve reductions in emissions at the lowest possible cost.

In each of the following cases, do you think the price elasticity of supply is (i) perfectly elastic; (ii) perfectly inelastic; (iii) elastic, but not perfectly elastic; or (iv) inelastic, but not perfectly inelastic? Explain using a diagram. An increase in demand this summer for luxury cruises leads to a huge jump in the sales price of a cabin on the Queen Mary 2. The price of a kilowatt of electricity is the same during periods of high electricity demand as during periods of low electricity demand. Fewer people want to fly during February than during any other month. The airlines cancel about 10% of their flights as ticket prices fall about 20% during this month.

a. Supply is perfectly inelastic: the quantity of cabins on the Queen Mary 2 is fixed. As demand increases (a rightward shift in the demand curve), the price of a cabin on the Queen Mary 2 increases, without an increase in the quantity supplied. b. Supply is perfectly elastic. As demand changes (for instance, as demand increases in times of high electricity demand), price does not change but the quantity supplied does change. c. Supply is inelastic. As price falls by 20%, the quantity supplied falls by 10%. This implies a price elasticity of supply of 10%/20% = 0.5 which is inelastic. d. Supply is elastic. As price falls by 30%, the quantity supplied falls by more than 50%. This implies a price elasticity of supply greater than 50%/30% = 1.7.

Assume that the demand for gasoline is inelastic and supply is relatively elastic. The government imposes a sales tax on gasoline. The tax revenue is used to fund research into clean fuel alternatives to gasoline, which will improve the air we all breathe. Who bears more of the burden of this tax, consumers or producers? Show in a diagram who bears how much of the burden. Is this tax based on the benefits principle or the ability-to-pay principle? Explain.

a. The accompanying diagram shows an inelastic (relatively steep) demand curve for gasoline. The tax, whether imposed on consumers or producers, drives a wedge between the price paid by consumers, PC , and the price received by producers, PP . The burden of the tax is illustrated by the gray and light gray areas A and B. Area A is the loss of consumer surplus—the burden of the tax that falls on consumers. Area B is the loss of producer surplus—the burden of the tax that falls on produc- ers. Here, the burden is borne predominantly by consumers. b. Consumers who drive cars that use more gasoline pay most of the tax. If this tax were based on the benefits principle, the tax revenue would have to benefit those who pay most of the tax. However, everyone benefits equally from research into cleaner fuel, because of the improvement in air quality, so the tax is not based on the benefits principle. If this tax were based on the ability-to-pay principle, more of the tax would have to be paid by those who have a greater ability to pay. This would be true if cars that consume more gasoline were driven largely by higher- income individuals. To some extent this may be true: higher-income individuals drive larger cars, SUVs, and so on, and to the extent to which this is true, the tax is based on the ability-to-pay principle. However, a significant number of lower- income individuals drive old, fuel-inefficient cars. To the extent to which this is true, the tax is not based on the ability-to-pay principle.

Assume De Beers is the sole producer of diamonds. When it wants to sell more diamonds, it must lower its price in order to induce shoppers to buy more. Furthermore, each additional diamond that is produced costs more than the previous one due to the difficulty of mining for diamonds. De Beers's total benefit schedule is given in the accompanying table, along with its total cost schedule. Draw the marginal cost curve and the marginal benefit curve and, from your diagram, graphically derive the optimal quantity of diamonds to produce. Calculate the total profit to De Beers from producing each quantity of diamonds. Which quantity gives De Beers the highest total profit?

a. The accompanying table shows the marginal benefit and marginal cost of each diamond. The accompanying diagram graphs marginal benefit and marginal cost. From the diagram, you can conclude that the optimal number of diamonds to produce is 5. b.The accompanying table calculates the total net gain to De Beers from producing each quantity of diamonds. The quantity that gives De Beers the greatest total net gain is 5 diamonds. This is, of course, just what you found in part a.

Marty's Frozen Yogurt is a small shop that sells cups of frozen yogurt in a university town. Marty owns three frozen-yogurt machines. His other inputs are refrigerators, frozen-yogurt mix, cups, sprinkle toppings, and, of course, workers. He estimates that his daily production function when he varies the number of workers employed (and at the same time, of course, yogurt mix, cups, and so on) is as shown in the accompanying table. What are the fixed inputs and variable inputs in the production of cups of frozen yogurt? Draw the total product curve. Put the quantity of labor on the horizontal axis and the quantity of frozen yogurt on the vertical axis. What is the marginal product of the first worker? The second worker? The third worker? Why does marginal product decline as the number of workers increases?

a. The fixed inputs are those whose quantities do not change as the quantity of out- put changes: frozen-yogurt machines, refrigerators, and the shop. The variable inputs are those whose quantities do change as the quantity of output changes: frozen-yogurt mix, cups, sprinkle toppings, and workers. b. The accompanying diagram illustrates the total product curve. (increasing curve the flatten) c. The marginal product, MPL, of the first worker is 110 cups. The MPL of the sec- ond worker is 90 cups. The MPL of the third worker is 70 cups. The MPL of labor declines as more and more workers are added due to the principle of diminishing returns to labor. Since the number of frozen-yogurt machines is fixed, as workers are added there are fewer and fewer machines for each worker to work with, mak- ing each additional worker less and less productive.

Do you think the price elasticity of demand for Ford sport-utility vehicles (SUVs) will increase, decrease, or remain the same when each of the following events occurs? Explain your answer. Other car manufacturers, such as General Motors, decide to make and sell SUVs. SUVs produced in foreign countries are banned from the American market. Due to ad campaigns, Americans believe that SUVs are much safer than ordinary passenger cars. The time period over which you measure the elasticity lengthens. During that longer time, new models such as four-wheel-drive cargo vans appear.

a. The price elasticity of demand for Ford SUVs will increase because more substitutes are available. b. The price elasticity of demand for Ford SUVs will decrease because fewer substitutes are available. c. The price elasticity of demand for Ford SUVs will decrease because other cars are viewed as less of a substitute. d. The price elasticity of demand for Ford SUVs will increase over time because more substitutes (such as four-wheel-drive cargo vans) become available.

In 1990, the United States began to levy a tax on sales of luxury cars. For simplicity, assume that the tax was an excise tax of $6,000 per car. The accompanying figure shows hypothetical demand and supply curves for luxury cars. Under the tax, what is the price paid by consumers? What is the price received by producers? What is the government tax revenue from the excise tax? Over time, the tax on luxury automobiles was slowly phased out (and completely eliminated in 2002). Suppose that the excise tax falls from $6,000 per car to $4,500 per car. After the reduction in the excise tax from $6,000 to $4,500 per car, what is the price paid by consumers? What is the price received by producers? What is tax revenue now? Compare the tax revenue created by the taxes in parts a and b. What accounts for the change in tax revenue from the reduction in the excise tax?

a. The price paid by consumers is $54,000. The price received by producers is $48,000. The government's tax revenue is $6,000 per car × 40,000 cars = $240 million. b. The price paid by consumers is now $53,000. The price received by producers is $48,500. The government's tax revenue is $4,500 per car × 60,000 cars = $270 million. c. The government tax revenue rose as a result of the reduction in the excise tax. This occurs because the supply of and the demand for luxury automobiles are both highly elastic: a fall in the price paid by consumers leads to a large increase in the quantity demanded, and a rise in the price received by producers leads to a large increase in the quantity supplied. As a result, reducing the tax leads to a large increase in the quantity of luxury automobiles bought and sold—so large, in fact, that the increase in the quantity bought and sold more than makes up for the decrease in the tax per car.

You are advising the government on how to pay for national defense. There are two proposals for a tax system to fund national defense. Under both proposals, the tax base is an individual's income. Under proposal A, all citizens pay exactly the same lump-sum tax, regardless of income. Under proposal B, individuals with higher incomes pay a greater proportion of their income in taxes. Is the tax in proposal A progressive, proportional, or regressive? What about the tax in proposal B? Is the tax in proposal A based on the ability-to-pay principle or on the benefits principle? What about the tax in proposal B? In terms of efficiency, which tax is better? Explain.

a. The tax in proposal A is regressive: since everyone pays the same dollar amount in taxes, higher-income individuals pay a lower percentage of their income in taxes. The tax in proposal B is progressive: higher-income individuals pay a higher per- centage of their income in taxes. b. Every citizen benefits equally from national defense. The tax in proposal A is based on the benefits principle: since everyone benefits equally, everyone pays equally. The tax in proposal B is based on the ability-to-pay principle: higher-income indi- viduals are able to pay more taxes, and under this proposal they do pay more taxes. c. In terms of efficiency, the lump-sum tax is better. The lump-sum tax creates a marginal tax rate of zero: once the tax is paid, every additional dollar earned is no longer taxed. Since it does not depend on people's actions (how much income they choose to earn), it does not distort their incentives to earn income. The tax in proposal B has a higher marginal tax rate: each additional dollar of income earned is taxed. And the marginal tax rate is increasing: each additional dollar of income earned is taxed more than the previous dollar. This creates inefficiency, because it distorts the incentive to earn more income.

Which of the following are characterized by network externalities? Which are not? Explain. The choice between installing 110-volt electrical current in structures rather than 220-volt The choice between purchasing a Toyota versus a Ford The choice of a printer, where each printer requires its own specific type of ink cartridge The choice of whether to purchase an iPod Touch or an iPod Nano.

a. This choice is characterized by a network externality:you want to use the standard that is more widely adopted by other people. That's because the greater the use of a given electricity standard, the greater the availability of appliances and other electrical items using that standard. b. This choice is not characterized by a network externality because there are mil- lions of Fords and Toyotas already on the road. Consequently, you will easily find a Toyota or Ford repair shop whenever you need one. There would be a network externality present in the choice of buying a new make of car in which you could not be assured of finding a mechanic for it. c. This choice is characterized by a network externality because you want to be assured of finding a cartridge for your printer in the future. If few people use your type of printer, you may be unable to find a cartridge; if many people use the printer, you are much more likely to find a cartridge for it. d. This choice is not characterized by a network externality because an iPod Touch and an iPod Nano use the same operating system. Any song that you could down- load using an iPod Touch you could also download using an iPod Nano. Any repair shop that can repair an iPod Touch can also repair an iPod Nano.

Bruno can spend his income on two different goods: Beyoncé MP3s and notebooks for his class notes. For each of the following three situations, decide if the given consumption bundle is within Bruno's consumption possibilities. Then decide if it lies on the budget line or not. MP3s cost $2 each, and notebooks cost $3 each. Bruno has income of $60. He is considering a consumption bundle containing 15 MP3s and 10 notebooks. MP3s cost $2 each, and notebooks cost $5 each. Bruno has income of $110. He is considering a consumption bundle containing 20 MP3s and 10 notebooks. MP3s cost $3 each, and notebooks cost $10 each. Bruno has income of $50. He is considering a consumption bundle containing 10 MP3s and 3 notebooks.

a. This consumption bundle costs $10 × 3 + $2 × 15 = $60, which is exactly equal to Bruno's income of $60. So the bundle is within Bruno's consumption possibilities. And, since he spends all his money, it lies on his budget line. b. This consumption bundle costs $10 × 3 + $5 × 10 = $80, which is less than Bruno's income of $110. So the bundle is within Bruno's consumption possibili- ties. However, since he does not spend all his money, it does not lie on his budget line; it lies below his budget line. c. This consumption bundle costs $20 × 2 + $10 × 2 = $60, which is more than Bruno's income of $50. So the bundle is not within Bruno's consumption possi- bilities; it lies above his budget line.

In the United States, 2013 was a bad year for growing wheat. And as wheat supply decreased, the price of wheat rose dramatically, leading to a lower quantity demanded (a movement along the demand curve). The accompanying table describes what happened to prices and the quantity of wheat demanded. Using the midpoint method, calculate the price elasticity of demand for winter wheat. What is the total revenue for U.S. wheat farmers in 2012 and 2013? Did the bad harvest increase or decrease the total revenue of U.S. wheat farmers? How could you have predicted this from your answer to part a?

a. Using the midpoint method, the percent change in the quantity of U.S. winter wheat demanded is 2.0 billion − 2.2 billion × 100 = −0.2 billion × 100 = −9.5% 2.1 billion 2.1 billion and the percent change in the price of U.S. winter wheat is $4.26 − $3.42 × 100 = $0.84 × 100 = 21.9% $3.84 $3.84 Dropping the minus sign, the price elasticity of demand is therefore 9.5% = 0.43 21.9% so that demand is inelastic. b. The total revenue in 2006 is the price per bushel in 2006 times the quantity of bush- els demanded in 2006. That is, total revenue in 2006 is $3.42 × 2.2 billion = $7.524 billion. Similarly, total revenue in 2007 is $4.26 × 2.0 billion = $8.52 billion. c. The rise in price from 2006 to 2007 increased U.S. wheat farmers' total revenue. This could have been predicted by knowing that demand is inelastic: in part a we calculated a price elasticity of demand of 0.43. The price effect of this price rise (which tends to increase total revenue) outweighed the quantity effect (which tends to decrease total revenue).

Patty delivers pizza using her own car, and she is paid according to the number of pizzas she delivers. The accompanying table shows Patty's total benefit and total cost when she works a specific number of hours. Use marginal analysis to determine Patty's optimal number of hours worked. Calculate the total profit to Patty from working 0 hours, 1 hour, 2 hours, and so on. Now suppose Patty chooses to work for 1 hour. Compare her total profit from working for 1 hour with her total profit from working the optimal number of hours. How much would she lose by working for only 1 hour?

a. We first need to work out Patty's marginal benefit and marginal cost of each addi- tional hour worked, which are shown in the accompanying table. For instance, as Patty increases the number of hours worked from 2 to 3, her benefit increases from $55 to $75; that is, her marginal benefit is $20. Similarly, as she increases the number of hours worked from 2 to 3, her cost increases from $21 to $34; that is, her marginal cost is $13. Repeating this for increases in the number of hours from 0 to 1, from 1 to 2, and so on, gives the data in the accompanying table. Patty should work for 3 hours because her marginal benefit of going from 2 hours to 3 hours ($20) exceeds the marginal cost of going from 2 hours to 3 hours ($13). But if she went from 3 hours to 4 hours, Patty's marginal cost ($16) would exceed her marginal benefit ($15). So working that fourth hour is not optimal. b. Patty's loss from working for only 1 hour instead of the optimal 3 hours is $41 − $20 = $21.

Suppose you have just paid a nonrefundable fee of $1,000 for your meal plan for this academic term. This allows you to eat dinner in the cafeteria every evening. You are offered a part-time job in a restaurant where you can eat for free each evening. Your parents say that you should eat dinner in the cafeteria anyway, since you have already paid for those meals. Are your parents right? Explain why or why not. You are offered a part-time job in a different restaurant where, rather than being able to eat for free, you receive only a large discount on your meals. Each meal there will cost you $2; if you eat there each evening this semester, it will add up to $200. Your roommate says that you should eat in the restaurant since it costs less than the $1,000 that you paid for the meal plan. Is your roommate right? Explain why or why not.

a. Your parents are wrong. They are making the mistake of considering sunk costs. Since the $1,000 that you have already paid for the meal plan is nonrefundable, it should not enter into your decision making now. Your decision of where to eat should depend only on those costs and benefits that are affected by your decision. Since both the cafeteria meals and the restaurant meals are free, you should choose to eat where the benefit to you (convenience, quality of food, and so on) is greater. b. Your roommate is wrong. Since the $1,000 that you have already paid for the meal plan is nonrefundable, it should not enter into your decision making now. It is a sunk cost. In deciding where to eat, you should weigh the benefit and cost of eating in the restaurant (where each meal costs $2) against the benefit and cost of eating in the cafeteria (where meals are free). You may decide to eat in the restaurant, but only if that gives you a benefit that is at least $2 greater than the benefit you get from eating in the cafeteria.

The United States imposes an excise tax on the sale of domestic airline tickets. Let's assume that in 2013 the total excise tax was $6.10 per airline ticket (consisting of the $3.60 flight segment tax plus the $2.50 September 11 fee). According to data from the Bureau of Transportation Statistics, in 2013, 643 million passengers traveled on domestic airline trips at an average price of $380 per trip. The accompanying table shows the supply and demand schedules for airline trips. The quantity demanded at the average price of $380 is actual data; the rest is hypothetical. What is the government tax revenue in 2013 from the excise tax? On January 1, 2014, the total excise tax increased to $6.20 per ticket. What is the quantity of tickets transacted now? What is the average ticket price now? What is the 2014 government tax revenue? Does this increase in the excise tax increase or decrease government tax revenue?

a.Tax revenue is $6.10 per trip × 630 million trips = $3,843 million. b. The equilibrium quantity now falls to 629 million, with the price rising to $337.02. Tax revenue rises to $6.20 per trip × 629 million trips = $3,899.80 million. c. The increase in the excise tax increases government tax revenue.

In each of the following cases involving taxes, explain: (i) whether the incidence of the tax falls more heavily on consumers or producers, (ii) why government revenue raised from the tax is not a good indicator of the true cost of the tax, and (iii) how deadweight loss arises as a result of the tax. The government imposes an excise tax on the sale of all college textbooks. Before the tax was imposed, 1 million textbooks were sold every year at a price of $50. After the tax is imposed, 600,000 books are sold yearly; students pay $55 per book, $30 of which publishers receive. The government imposes an excise tax on the sale of all airline tickets. Before the tax was imposed, 3 million airline tickets were sold every year at a price of $500. After the tax is imposed, 1.5 million tickets are sold yearly; travelers pay $550 per ticket, $450 of which the airlines receive. The government imposes an excise tax on the sale of all toothbrushes. Before the tax, 2 million toothbrushes were sold every year at a price of $1.50. After the tax is imposed, 800,000 toothbrushes are sold every year; consumers pay $2 per toothbrush, $1.25 of which producers receive.

a.uAfteirtohenimpositionofthetax,consumerspay$5moreperbookthanbefore; publishers receive $20 less per book than before. Producers (publishers) bear more of the tax. The tax is $55 − $30 = $25 per book, and 600,000 books are bought and sold. So government revenue is $15 million. This, however, is a poor estimate of the cost of the tax, since it does not take into account the fact that, in addition to the higher price, there are now 400,000 potential consumers who would have bought the books without the tax but no longer will buy them. Deadweight loss arises because consumers and producers lose surplus that is not captured as gov- ernment revenue. That loss in surplus is accounted for by the 400,000 potential consumers and publishers who would have made transactions without the tax but do not once the tax is levied. b. After the imposition of the tax, travelers pay $50 more per ticket than before; air- lines receive $50 less than before. The tax is split evenly between consumers and producers. The tax is $550 − $450 = $100 per ticket, and 1.5 million tickets are bought and sold. So government revenue is $150 million. This, however, is a poor estimate of the cost of the tax, since it does not take into account the fact that, in addition to 1.5 million travelers paying higher prices, there are now 1.5 million potential consumers who would have bought tickets without the tax but no longer buy tickets. Deadweight loss arises because consumers and producers lose surplus that is not captured as government revenue. That loss in surplus is represented by the 1.5 million tickets that would have been transacted at the pre-tax price but are not transacted once the tax is levied. c. After the imposition of the tax, consumers pay $0.50 more per toothbrush than before; producers receive $0.25 less than before. The incidence of the tax falls mainly on consumers. The tax is $2.00 − $1.25 = $0.75 per toothbrush, and 800,000 toothbrushes are bought and sold. So government revenue is $600,000. This, however, is a poor estimate of the cost of the tax, since it does not take into account the fact that, in addition to 800,000 toothbrushes now being more expen- sive, there are 1.2 million toothbrushes that would have been transacted without the tax but are no longer transacted. Inefficiency arises because consumers and producers lose surplus that is not captured as government revenue. That loss in surplus is represented by the 1.2 million toothbrushes that would have been trans- acted at the pre-tax price but are not transacted once the tax is levied.

The Centers for Disease Control and Prevention (CDC) recommended against vaccinating the whole population against the smallpox virus because the vaccination has undesirable, and sometimes fatal, side effects. Suppose the accompanying table gives the data that are available about the effects of a smallpox vaccination program. Percent of population vaccinated Deaths due to smallpox Deaths due to vaccination side effects Calculate the marginal benefit (in terms of lives saved) and the marginal cost (in terms of lives lost) of each 10% increment of smallpox vaccination. Calculate the net increase in human lives for each 10% increment in population vaccinated. Using marginal analysis, determine the optimal percentage of the population that should be vaccinated.

a.uThte iacocomnpanying table gives the marginal benefit and the marginal cost of small- pox vaccination. The marginal benefit is the additional number of lives saved if we vaccinate an additional 10% of the population. For instance, if instead of vacci- nating 0% of the population (resulting in 200 deaths from smallpox), we vacci- nate 10% of the population (resulting in 180 deaths from smallpox), we have saved 20 lives. That is, the marginal benefit of vaccinating 10% (instead of 0%) of the population is 20 lives. Repeating this for the step from 10% to 20% vaccina- tion, and so on, gives us the marginal benefit numbers in the table. The marginal cost is the additional number of lives lost if we vaccinate an additional 10% of the population. For instance, if instead of vaccinating 0% of the population (resulting in 0 deaths due to side effects), we vaccinate 10% of the population (resulting in 4 deaths due to side effects), we have lost 4 lives. That is, the marginal cost of vac- cinating 10% (instead of 0%) of the population is 4 lives. Repeating this for the step from 10% to 20% vaccination, and so on, gives us the marginal cost numbers in the table. b. The optimal percentage of the population that should be vaccinated is 50%. Suppose we were vaccinating 40% of the population. Then vaccinating an addi- tional 10% (to bring the total up to 50%) would give us a marginal benefit of 20 lives saved. And vaccinating that additional 10% would give us a marginal cost of 17 lives lost. Since the marginal benefit exceeds the marginal cost, we do indeed want to vaccinate that additional 10% of the population. But do we want to go beyond 50% vaccination? Vaccinating an additional 10% (to bring the total up to 60%) would result in a marginal benefit of 20 lives saved and a marginal cost of 24 lives lost due to side effects. Since the marginal cost exceeds the marginal ben- efit, we do not want to increase the vaccination rate from 50% to 60%.

Cal "Cool" Cooper has $200 to spend on cell phones and sunglasses. Each cell phone costs $100 and each pair of sunglasses costs $50. Which bundles lie on Cal's budget line? Draw a diagram like Figure 10-4 in which both the marginal utility per dollar spent on cell phones and the marginal utility per dollar spent on sunglasses are illustrated. Use this diagram and the optimal consumption rule to decide how Cal should allocate his money. That is, from all the bundles on his budget line, which bundle will Cal choose? The accompanying table gives his utility of cell phones and sunglasses. The price of cell phones falls to $50 each, but the price of sunglasses remains at $50 per pair. Which bundles lie on Cal's budget line? Draw a diagram like Figure 10-4 in which both the marginal utility per dollar spent on cell phones and the marginal utility per dollar spent on sunglasses are illustrated. Use this diagram and the utility-maximizing principle of marginal analysis to decide how Cal should allocate his money. That is, from all the bundles on his budget line, which bundle will Cal choose? The accompanying table gives his utility of cell phones and sunglasses. How does Cal's consumption of cell phones change as the price of cell phones falls? In words, describe the income effect and the substitution effect of this fall in the price of cell phones, assuming that cell phones are a normal good.

a.uThte ifoollowning bundles lie on Cal's budget line: 0 cell phones, 4 pairs of sunglasses 1 cell phone, 2 pairs of sunglasses 2 cell phones, 0 pairs of sunglasses Going from 0 cell phones to 1 cell phone, the marginal utility per cell phone is 400 utils; that is, the marginal utility per dollar spent on cell phones is 4 utils. Going from 1 cell phone to 2 cell phones, the marginal utility per cell phone is 300 utils; that is, the marginal utility per dollar spent on cell phones is 3 utils. Going from 0 pairs of sunglasses to 2 pairs of sunglasses, the marginal utility per pair is 600/2 = 300 utils; that is, the marginal utility per dollar spent on sunglass- es is 6 utils. Going from 2 pairs of sunglasses to 4 pairs, the marginal utility per pair is 100/2 = 50 utils; that is, the marginal utility per dollar spent on sunglasses is 1 util. The marginal utility per dollar spent on cell phones and the marginal utility per dollar spent on sunglasses are plotted in the accompanying diagram.Of all the possible bundles Cal could consume (that is, from all the bundles on his budget line), the bundle that contains 1 cell phone and 2 pairs of sunglasses is optimal. At that bundle, the marginal utility per dollar spent on cell phones and the marginal utility per dollar spent on sunglasses are equal. By the optimal con- sumption rule, this is Cal's optimal consumption bundle. b. The bundles that lie on Cal's budget line are: 0 cell phones, 4 pairs of sunglasses 1 cell phone, 3 pairs of sunglasses 2 cell phones, 2 pairs of sunglasses 3 cell phones, 1 pair of sunglasses 4 cell phones, 0 pairs of sunglasses The accompanying table calculates marginal utility per cell phone, marginal utility per pair of sunglasses, marginal utility per dollar spent on cell phones, and marginal utility per dollar spent on sunglasses.From all the bundles on Cal's budget line, the marginal utility per dollar spent on cell phones is the same as the marginal utility per dollar spent on sunglasses at 2 cell phones and 2 pairs of sunglasses. By the optimal consumption rule, this is Cal's optimal consumption bundle. c. Cal's consumption of cell phones increases from 1 to 2 as the price of cell phones falls. This is due to two effects. The substitution effect says that as the price of cell phones falls, their opportunity cost falls: Cal now has to give up fewer pairs of sunglasses for 1 cell phone. This makes cell phones more attractive, and Cal sub- stitutes cell phones in place of sunglasses. The income effect says that as cell phones become cheaper, Cal gets richer in a real sense: his income now buys more goods. Since cell phones are a normal good, when the purchasing power of Cal's income rises, he consumes more cell phones. Both effects contribute to the fact that as the price of cell phones falls, Cal's consumption of cell phones increases.

Amy, Bill, and Carla all mow lawns for money. Each of them operates a different lawn mower. The accompanying table shows the total cost to Amy, Bill, and Carla of mowing lawns. Calculate Amy's, Bill's, and Carla's marginal costs, and draw each of their marginal cost curves. Who has increasing marginal cost, who has decreasing marginal cost, and who has constant marginal cost?

https://people.ucsc.edu/~nuclear/econ1/testinfo/Krugman2e_Solutions_CH09.pdf (#6)

The production function for Marty's Frozen Yogurt is given in Problem 2. Marty pays each of his workers $80 per day. The cost of his other variable inputs is $0.50 per cup of yogurt. His fixed cost is $100 per day. What is Marty's variable cost and total cost when he produces 110 cups of yogurt? 200 cups? Calculate variable and total cost for every level of output given in Problem 2. Draw Marty's variable cost curve. On the same diagram, draw his total cost curve. What is the marginal cost per cup for the first 110 cups of yogurt? For the next 90 cups? Calculate the marginal cost for all remaining levels of output.

https://people.ucsc.edu/~nuclear/econ1/testinfo/Krugman2e_Solutions_CH12.pdf (#3)


Set pelajaran terkait

Eight Unique Features of E-commerce

View Set

Algorithms Final Exam ORDER OF MAGNITUDE IS IN SQUARE BRACKETS LIKE O[n]

View Set

Fluid and Electrolytes (Fluid volume deficit)

View Set

Quiz Block 4: PUNCTUATION/MECHANICS: Commas I, paragraph editing

View Set

Environmental earth science final

View Set

Hesi Final Study Questions: Med Surg II

View Set